Skip to content

Commit 7a6bad5

Browse files
committed
feat: Update chap4.
1 parent dd15c77 commit 7a6bad5

File tree

2 files changed

+34
-34
lines changed

2 files changed

+34
-34
lines changed

chap4/part1.tex

Lines changed: 3 additions & 3 deletions
Original file line numberDiff line numberDiff line change
@@ -81,7 +81,7 @@ \subsection{滤波器的表示}
8181
其中 $h(n)$ 为系统的单位脉冲响应,$P$ 为一个常数。
8282
\end{exercise}
8383

84-
\begin{exercise}
84+
\begin{homework}
8585
已知某系统的差分方程如下式:
8686
\begin{align*}
8787
y(n) = x(n) + x(n - 3) + 0.7y(n - 1) + 0.6y(n - 2).
@@ -90,7 +90,7 @@ \subsection{滤波器的表示}
9090
\item 判断系统的脉冲响应类型。
9191
\item 画出该系统的信号流图。
9292
\end{enumerate}
93-
\end{exercise}
93+
\end{homework}
9494

9595
\begin{solution}
9696
\begin{enumerate}[label=(\arabic*)]
@@ -140,7 +140,7 @@ \subsection{滤波器的表示}
140140
\path [line] (zy2) |- (zgy2);
141141
\path [line] (zgy2.west) -- (sum);
142142
\end{tikzpicture}
143-
\caption{信号流图}
143+
\caption{作业 \thehomework~ 的信号流图}
144144
\label{fig:signal_flow_diagram}
145145
\end{figure}
146146
\end{enumerate}

chap4/part2.tex

Lines changed: 31 additions & 31 deletions
Original file line numberDiff line numberDiff line change
@@ -185,7 +185,7 @@ \subsubsection{Z 变换的收敛域}
185185
\subsubsection{常见序列及其 ZT}
186186

187187
\begin{example}[单位冲激序列的 ZT]
188-
单位冲击序列 $\delta(n)$ 的 Z 变换为
188+
单位冲激序列 $\delta(n)$ 的 Z 变换为
189189
\begin{align*}
190190
\mathcal{Z}[\delta(n)] = 1, \quad (\text{ROC}: 0 \le \abs{z} \le +\infty).
191191
\end{align*}
@@ -463,13 +463,13 @@ \subsubsection{逆 Z 变换的求解}
463463
再使用 Z 变换的\bd{时移性质}得到 $x(n)$
464464
\end{enumerate}
465465

466-
\begin{example}
466+
\begin{exercise}
467467
已知
468468
\begin{align*}
469469
X(z) = \frac{6 + z^{-5}}{1 - 0.25z^{-2}},
470470
\end{align*}
471471
求对应的因果序列。
472-
\end{example}
472+
\end{exercise}
473473

474474
\begin{solution}
475475
(Remove-Restore 法)
@@ -502,13 +502,13 @@ \subsubsection{逆 Z 变换的求解}
502502
\end{align*}
503503
\end{solution}
504504

505-
\begin{example}
505+
\begin{exercise}
506506
已知
507507
\begin{align*}
508508
X(z) = \frac{7 - 9.5z^{-1} - 3.5z^{-2} + 5.5z^{-3}}{(1 - z^{-2})(1 - 0.5z^{-1})(1 - 1.5z^{-1})},
509509
\end{align*}
510510
求其对应的所有可能的序列。
511-
\end{example}
511+
\end{exercise}
512512

513513
\begin{solution}
514514
由于
@@ -628,7 +628,7 @@ \subsubsection{传递函数与系统的串并联}
628628
\end{align*}
629629
\end{solution}
630630

631-
\begin{example}
631+
\begin{exercise}
632632
已知某滤波器的传递函数如下式:
633633
\begin{align*}
634634
H(z) = \frac{2 - 3z^{-1} + 4z^{-3}}{1 + 0.2z^{-1} - 0.3z^{-2} + 0.5z^{-4}}.
@@ -637,7 +637,7 @@ \subsubsection{传递函数与系统的串并联}
637637
\item 写出相应的差分方程。
638638
\item 画出滤波器的信号流程图。
639639
\end{enumerate}
640-
\end{example}
640+
\end{exercise}
641641

642642
\begin{solution}
643643
\begin{enumerate}[label=(\arabic*)]
@@ -702,19 +702,19 @@ \subsubsection{传递函数与系统的串并联}
702702
\path [line] (zy3) |- (zgy3);
703703
\path [line] (zgy3.west) -- (sum);
704704
\end{tikzpicture}
705-
\caption{\theexample~ 的信号流图}
705+
\caption{习题 \theexercise~ 的信号流图}
706706
\label{fig:chap4-part2-quiz3}
707707
\end{figure}
708708
\end{enumerate}
709709
\end{solution}
710710

711-
\begin{example}
711+
\begin{exercise}
712712
已知滤波器的差分方程为
713713
\begin{align*}
714714
y(n) + 0.8y(n - 1) - 0.9y(n - 2) = x(n - 2),
715715
\end{align*}
716716
求该滤波器的传递函数(频率响应)。若已知其为因果系统,求其收敛域。
717-
\end{example}
717+
\end{exercise}
718718

719719
\begin{solution}
720720
由差分方程可得其传递函数为
@@ -731,12 +731,12 @@ \subsubsection{传递函数与系统的串并联}
731731
\end{align*}
732732
\end{solution}
733733

734-
\begin{example}
734+
\begin{exercise}
735735
对下面给出的 Z 变换结果,求它对应的序列。
736736
\begin{align*}
737737
X(z) = \frac{2z^{-1} - z^{-2}}{1 - 1.6z^{-1} - 0.8z^{-2}}.
738738
\end{align*}
739-
\end{example}
739+
\end{exercise}
740740

741741
\begin{solution}
742742
由题知
@@ -833,7 +833,7 @@ \subsubsection{传递函数与系统的串并联}
833833
\path [line] (zy2) |- (zgy2);
834834
\path [line] (zgy2.west) -- (sum);
835835
\end{tikzpicture}
836-
\caption{\theexample~ 的直接 I 型实现}
836+
\caption{习题 \theexercise~ 的直接 I 型实现}
837837
\label{fig:chap4-part2-quiz6-1}
838838
\end{figure}
839839

@@ -880,7 +880,7 @@ \subsubsection{传递函数与系统的串并联}
880880
\path [line] (circ2) -- (zgy2);
881881
\path [line] (zgy2.west) -- (sumy);
882882
\end{tikzpicture}
883-
\caption{\theexample~ 的直接 II 型实现}
883+
\caption{习题 \theexercise~ 的直接 II 型实现}
884884
\label{fig:chap4-part2-quiz6-2}
885885
\end{figure}
886886
\item$H(z)$ 化简可得
@@ -895,15 +895,15 @@ \subsubsection{传递函数与系统的串并联}
895895
\end{enumerate}
896896
\end{solution}
897897

898-
\begin{exercise}
898+
\begin{homework}
899899
用直接 I 型和直接 II 型(标准型)结构实现以下系统函数:
900900
\begin{align*}
901901
H(z) = \frac{3 + 4.2z^{-1}+0.8z^{-2}}{2 + 0.6z^{-1} - 0.4z^{-2}}.
902902
\end{align*}
903-
\end{exercise}
903+
\end{homework}
904904

905905
\begin{solution}
906-
直接 I 型结构如图 \ref{fig:chap4-part2-exercise1-1} 所示,直接 II 型结构如图 \ref{fig:chap4-part2-exercise1-2} 所示。
906+
直接 I 型结构如图 \ref{fig:chap4-part2-homework1-1} 所示,直接 II 型结构如图 \ref{fig:chap4-part2-homework1-2} 所示。
907907
\begin{figure}[H]
908908
\centering
909909
\tikzstyle{block} = [draw, rectangle, minimum height=1cm, minimum width=1cm]
@@ -952,8 +952,8 @@ \subsubsection{传递函数与系统的串并联}
952952
\path [line] (zy2) |- (zgy2);
953953
\path [line] (zgy2.west) -- (sum);
954954
\end{tikzpicture}
955-
\caption{习题 \theexercise~ 的直接 I 型结构}
956-
\label{fig:chap4-part2-exercise1-1}
955+
\caption{作业 \thehomework~ 的直接 I 型结构}
956+
\label{fig:chap4-part2-homework1-1}
957957
\end{figure}
958958

959959
\begin{figure}[H]
@@ -1000,12 +1000,12 @@ \subsubsection{传递函数与系统的串并联}
10001000
\path [line] (circ2) -- (zgy2);
10011001
\path [line] (zgy2.west) -- (sumy);
10021002
\end{tikzpicture}
1003-
\caption{习题 \theexercise~ 的直接 II 型结构}
1004-
\label{fig:chap4-part2-exercise1-2}
1003+
\caption{作业 \thehomework~ 的直接 II 型结构}
1004+
\label{fig:chap4-part2-homework1-2}
10051005
\end{figure}
10061006
\end{solution}
10071007

1008-
\begin{exercise}
1008+
\begin{homework}
10091009
已知某系统的差分方程如下式:
10101010
\begin{align*}
10111011
y(n) = 2x(n) + x(n - 3) - 0.9y(n - 1) + 0.36y(n - 2).
@@ -1015,15 +1015,15 @@ \subsubsection{传递函数与系统的串并联}
10151015
\item 画出该系统的信号流图。
10161016
\item 求该系统对应的因果序列及其收敛域,并判断该状态下系统是否稳定。
10171017
\end{enumerate}
1018-
\end{exercise}
1018+
\end{homework}
10191019

10201020
\begin{solution}
10211021
\begin{enumerate}[label=(\arabic*)]
10221022
\item 由差分方程可得其传递函数为
10231023
\begin{align*}
10241024
H(z) = \frac{2 + z^{-3}}{1 + 0.9z^{-1} - 0.36z^{-2}}.
10251025
\end{align*}
1026-
\item 画出该系统的信号流图如图 \ref{fig:chap4-part2-exercise2} 所示。
1026+
\item 画出该系统的信号流图如图 \ref{fig:chap4-part2-homework2} 所示。
10271027
\begin{figure}[H]
10281028
\centering
10291029
\tikzstyle{block} = [draw, rectangle, minimum height=1cm, minimum width=1cm]
@@ -1070,8 +1070,8 @@ \subsubsection{传递函数与系统的串并联}
10701070
\path [line] (zy2) |- (zgy2);
10711071
\path [line] (zgy2.west) -- (sum);
10721072
\end{tikzpicture}
1073-
\caption{习题 \theexercise~ (2) 的直接 I 型实现}
1074-
\label{fig:chap4-part2-exercise2}
1073+
\caption{作业 \thehomework~ (2) 的直接 I 型实现}
1074+
\label{fig:chap4-part2-homework2}
10751075
\end{figure}
10761076
\item 考虑 $H(z) = (2 + z^{-3})W(z)$,则
10771077
\begin{align*}
@@ -1102,7 +1102,7 @@ \subsubsection{传递函数与系统的串并联}
11021102
\end{enumerate}
11031103
\end{solution}
11041104

1105-
\begin{exercise}
1105+
\begin{homework}
11061106
已知离散系统的差分方程为
11071107
\begin{align*}
11081108
y(n) = x(n) + 4x(n - 1) + 0.7y(n - 1) - 0.1y(n - 2),
@@ -1115,7 +1115,7 @@ \subsubsection{传递函数与系统的串并联}
11151115
\item 说明系统的高低通特性。
11161116
\item 说明系统的稳定性。
11171117
\end{enumerate}
1118-
\end{exercise}
1118+
\end{homework}
11191119

11201120
\begin{solution}
11211121
\begin{enumerate}[label=(\arabic*)]
@@ -1146,7 +1146,7 @@ \subsubsection{传递函数与系统的串并联}
11461146
\abs{H(\omega)} & = \frac{\abs{1 + 4\mathe^{\mathi\omega}}}{\abs{1 - 0.7\mathe^{\mathi\omega} + 0.1\mathe^{2\mathi\omega}}} \\
11471147
& = \frac{\sqrt{17 + 8\cos\omega}}{\sqrt{1.5 + 0.2\cos2\omega - 1.54\cos\omega}}.
11481148
\end{align*}
1149-
注意到 $H(0) = 25/2, H(\pi) = 5/3$,故其系统幅频响应图如图 \ref{fig:chap4-part2-exercise3} 所示。
1149+
注意到 $H(0) = 25/2, H(\pi) = 5/3$,故其系统幅频响应图如图 \ref{fig:chap4-part2-homework3} 所示。
11501150
\begin{figure}[H]
11511151
\centering
11521152
\begin{tikzpicture}
@@ -1174,8 +1174,8 @@ \subsubsection{传递函数与系统的串并联}
11741174
\node at (axis cs:0, 1.667) [anchor=east] {$5/3$};
11751175
\end{axis}
11761176
\end{tikzpicture}
1177-
\caption{习题 \theexercise~ 的系统幅频响应图}
1178-
\label{fig:chap4-part2-exercise3}
1177+
\caption{作业 \thehomework~ 的系统幅频响应图}
1178+
\label{fig:chap4-part2-homework3}
11791179
\end{figure}
11801180
由图可知,该系统是低通滤波器。
11811181
\item 只有 ROC 为 $\abs{z} > 0.5$ 时,包含单位圆,该系统是稳定的;其余情况下系统是不稳定的。

0 commit comments

Comments
 (0)